Tuyển tập đề thi vô địch bất đẳng thức thế giới P5

17 332 4
Tuyển tập đề thi vô địch bất đẳng thức thế giới P5

Đang tải... (xem toàn văn)

Tài liệu hạn chế xem trước, để xem đầy đủ mời bạn chọn Tải xuống

Thông tin tài liệu

Old and New > toi a; Inequalities 111 and so nai TJ n—1 —_ mm a, Using this last inequality, it remains to | prove that +? Th a; +1 -S ny i=1 Tt đ | — @n41 nS i=1 Th =1 +đ„ +1 a, 71 ap —S a;+ (n— lay, + +1 —1 | - > Now, we will break this inequality into Gn+1 and (Tre +? +(n—-l)a)đxy 11 — đạiJ)>0 ?ì ben Tr — Sa) i=1 —An41 +1 ?ì s34 — Yer) i=1 > +1 Let us justify these two inequalities The first one is pretty obvious n n—L _ (n — 1)djp.1 — đu = II Io — „+1 + đn+1) + (nm — Yang, 21 +? —1 > Ong tangy: So (ai — Angi) +(n- n—-1 — any, = — Yan, - đa} =0 21 Now, let us prove the second inequality It can be written as nm Tr na; i=1 n Because n › 1=1 „+1 — nr À ad? n > dua nề ¿1 nr at n — À i=1 dị T,+—H i=1 n ae — › i=1 a~' > (using Chebyshev’s Inequality) and a,41 < —, it n is enough to prove that "nan + nr i=1 2+1 Tr + =1 yn but this one follows, because na??? + -a; n is proved 21 '> a; for all Thus, the inductive step 117 Prove that for any %1,%2, ,%n > with product 1, n SY) 1 2z [Som k=2 —Ín — va) n Clearly, we have x; < G and » x, > (n —1)G, so it suffices to prove that k=2 n n ono n (3) k=2 > 2G k=2 =.' k=2 We will prove that this inequality holds for all zs, ,„ > Because the inequality is homogeneous, it is enough to prove it when G = In this case, from the induction step we already have n n (ø—=2)3_z? +m—1> (yo) k=2 k=2 and so it suffices to prove that nm nm nm Soap tn-1>S 2x k=2 k=2 - k=2 (z¿T— 1)” >0, clearly true Thus, we have proved that ƒ(#1,#as, ,#„) < ƒ(zi,G,G, ,G) plete the inductive step, we will prove that ƒ(zi,G,G, ,ŒG) r= Gr- Now, to com- < Because clearly ;> the last assertion reduces to proving that (n — 1) ((n-6? which comes down to + aos) n—2 Gonz +n 7" = > ((m-)6+ a) \* 2n — "Gn=3 and this one is an immediate consequence of the AM-GM Inequality Old and New Inequalities 113 118 | Gabriel Dospinescu | Find the minimum value of the expression » where @1,@2, ,@n, < 4142 ay m - Aan - add up to and n > is an integer Solution: We will prove that the minimal value is equality, we find that nm (m — 1)Soa? =: Indeed, using Suranyi’s nh— nm nm +NQ1a2 4n > Soap =1 In- => NA, 42 4n > S =1 a7 (1 — (n — 1)aj) =1 Now, let us observe that ¬ ¬ az (1— (n= 1)ax) =_ _ (i) = (aoa) and so, by an immediate application of Holder Inequality, we have » n n—-1 ; a — k=1 fol ; 1— (n— 1)ag But for n > 3, we can apply Jensen’s Inequality to deduce that nooo ¬ — k=1 n > mr | Dae — k=1 — n ¬ ns Thus, combining all these inequalities, we have proved the inequality for n > For n = it reduces to proving that b Viner 2" which was proved in the solution of the problem 107 119 | Vasile Cirtoaje | Let a1,a2, ,a@, < be nonnegative real numbers such that Qa ajtaz+ +a2 ——————ccc n > v3 — 114 Solutions Prove that ay ag l-da 4 an la > na IlTa2 — l1—a2 Solution: We proceed by induction Clearly, the inequality is trivial for n= Now we suppose that the inequality is valid for n = k —1, k > and will prove that for We Qk Lae > T14? aj+as+ +a; ka —_— “1-4? Is a2 Tá IV Qy lay k assume, without loss of generality, that aj > a2 > > ax, therefore a > az Using the notation w —= ap +a? + -+ap_, ———c k—1 from a > a, it follows that « > a> ay a2 I-aP 1d ¬ From „3 _ ga - #8 we obtain đpy —1 Tạp 1—z2 ———- k—I1 ` › mm k-1 (k — 1)(z — a) — (k — 1)z 1—z2 ` — 1—a2 › a7 k—I1` (a — ay)(a + ax) z+ and Qk 1—a? ka? — a3 — Thus, by the inductive hypothesis, we have TH Tay ca + (k —1)a l-r —(a—ag)(1+aaz) (1 — az)(1 - a?) ka 1l-@ — Qk (ra a — x (= = (k-1)(a —a)(1 + az) _ a= je (1 — x?)(1 - a?) 1— a2 1-a? a -5) eee) (1 — x?)(x +a) _ (a — ag)(œ — a¿)[—1 + #Ÿ + d; + ray + a(2 + ay) + 07 + ara,(x + ag) + a 2d] (1— a3)(1— a¿)(1— #?)(œ + a) Since 2? — az = ka” — d2 k(a — ag)(a + ag) — dị —= , it follows that k—1 k-1 (a — ag)(@ — ag) = k(a — ag)? (a + ag) (k-1)(a@+a,z) — _ ` Old and New Inequalities 115 and hence we have to show that x? + az + rap + a(# + ag) + aŸ + aag(œ + ag) + a zag > In order to show this inequality, we notice that ka? 2) ka? + (k — 2)a2k wet dy = k— “k—1 and z-+døyg >\x/z2+a})>a — Consequently, we have b _ VWk_1 z + q2 + zay + a(# + ag) + 8Ÿ + azag(% + ag) + a2zag > (z2 + a2) + a(# + ag) + a2 > k >>(sẫ: | k TT atte > 3a — 1, and the proof is complete Equality holds when a, = ag = = Gn Remarks From the final solution, we can easily see that the inequality is valid for the larger condition 14+, /2+4 m+—1 This is the largest range for a, because Íor đị = đa = - = đ„ạ + = # and ø„ = — Ì (therefore a = #\/ * n ), from the given inequality we get a > The special case n = and a = v is a problem from Crux 2003 120 | Vasile Cirtoaje, Mircea Lascu | Let a,b,c, x,y,z be positive real numbers such that (atb+e\(e@tyt+z=(@P@4P4 ee)? +y42)=4 Prove that beryz A0CLY < J 9abczz(a + b+ e)(œ + + 2), or (ab + bc + ca)(xy + yz 4+ 2x) > 36abcryz (2) From (1) and (2), we conclude that < (ab + be + ca)(xy + yz + 2x) > 36abcryz, therefore < 36abcryz To have = 36abcryz, the equalities (ab + bc + ca)? = 3abe(a + b + c) and (cytyz+zx)* = 3xyz(x+y+z) are necessary But these conditions imply a = b = ¢ and x = y = 2, which contradict the relations (a +6+c¢)(x2x+y+2) = (a@+6%4 c2)(z? + + z?) = Thus, it follows that > 36abcryz 121 | Gabriel Dospinescu | For a given n > 2, find the minimal value of the constant k,, such that if 71, %2, ,%, > have product VI+kzạzi + + VJV1tknze woe 1, then + — VI + kn#„ Thus, we can find a number M and some numbers a; > which add up to 1, such that 1+ 2n — | „ (ø—1)?" = Mag >n-1 Old Thus, az and New Inequalities 117 i and we have HSS r (areag~')] > (wea)

Ngày đăng: 07/11/2013, 17:15

Từ khóa liên quan

Tài liệu cùng người dùng

Tài liệu liên quan